2014 dxdy logo

Научный форум dxdy

Математика, Физика, Computer Science, Machine Learning, LaTeX, Механика и Техника, Химия,
Биология и Медицина, Экономика и Финансовая Математика, Гуманитарные науки




Начать новую тему Ответить на тему На страницу Пред.  1 ... 4, 5, 6, 7, 8, 9, 10 ... 17  След.
 
 
Сообщение03.06.2006, 12:32 
Заблокирован


04/01/06

602
г. Москва, ФГУП НПО "Пульсар"
Шимпанзе писал(а):
Вот, пожалуй, практическое применение решения данной задачи: определить распределение линейной плотности заряда бесконечно тонкого янтарного ( или пластмассового) стержня, заряженного путем трения.
Задача все же физическая и мне не понятно, почему её стараются свести к чистой математике. Во-первых, это делает задачу принципиально неразрешимой ( возникает множество граничных эффектов) или, по крайней мере, с неудовлетворительным (или не полным) физическим решением.
Хотя бы то, что заряженная линейка имеет верхний предел энергии, да и вообще конкретную энергию следует из элементарной физической формулы.

W = q^2/2C,
где соответственно суммарный заряд и емкость линейки, которая зависит лишь от геометрической формы и размеров линейки.



Шимпанзе

1. Ну и чему, по Вашему, равна электрическая емкость бесконечно тонкого диэлектрического стержня конечной длины?
2. И чему равна величина энергии запасенной в такой емкости?
P.s.
Формулы штука удобная, только, что бы правильно ими пользоваться, надо знать их "происхождение", т.е. физику, т.к. у всякой формулы есть своя "область применения", определяемая все той же физикой.

 Профиль  
                  
 
 
Сообщение03.06.2006, 13:01 
Экс-модератор
Аватара пользователя


30/11/05
1275
незванный гость писал(а):
Аурелиано Буэндиа писал(а):
А у меня получилось строго $r_{min}>\frac{2a}{n^2}$
...
Я потом изложу свой подход подробно и свою точку зрения на предельный переход.


Я попросил у Вас обоснование. В ответ Вы привели вывод формулы "$r_{m}$ строго больше $\frac{a}{n(n-1)(\ln\frac{1}{2}(n-1)+1)}$". Возникает вопрос -- в чем новы Ваш способ и Ваша формула по сравнению с моим (ну не тем же, что вы называете равноотстоящие узлы эквидистантными, правда?)? Где вывод Вашей, более сильной формулы? Из каких соображений можно ее получить? (Замечу вскользь, что и в выводе Вашем ньюансы имеются... Энергия равноотстоящих зарядов равна $\frac{q^2}{a} \ n(n-1)\sum\limits_{k=2}^{n}\frac1k$. Так что детали Вашей оценки (типа 1/2 в логарифме) только вводят в заблуждение.)

Существование минимума ограниченной снизу функции на компакте мне казалось тривиальным. Стремление к 0 максимума расстояния между зарядами -- тоже, хотя я готов и пересмотреть эту оценку. У меня были кое-какие соображения, но может, мне и не удастся их формализовать на удовлетворяющем меня уровне. Тогда я признаю свою ошибку. Но и Вы до сих пор не изволили поместить свой подход на форуме.

Просто свист -- это кинуть формулу без обоснования (и даже без намека на то, из чего она получена). Если Вы будете настаивать, я готов привести еще по крайней мере один пример. Вам это надо?

Остановлюсь на нашем взаимопонимании. Я в первый раз сталкиваюсь с использованием интегралов для описания потенциала дискретных зарядов. В цитированном же Вами сообщении я рассматривал непрерывный разряд (отчасти именно под впечатлением от Вашего сообщения), на что и получил Вашу же отповедь. В которой Вы обещали свое решение потом. Интересно, почему? Вы пытаетесь помочь Mopnexу решить самому?

И последнее. Я с пользой потратил два часа на численные эксперименты. Я поделился своими результатами со всеми участниками. Но из чего Вы сделали вывод, что я использовал их для мотивации своих предшествующих утверждений, я ума не приложу. Мне казалось, я достаточно ясно объясняю, что есть доказательное утверждение, что гипотеза, а что -- результат эксперимента. Или Вам "теперь понятно" что-то другое?

Я все-таки склонен считать, что общение с целью решения задачи было бы более продуктивным, чем попытки пререканий """ты такая -- ты такой -- ты плохая -- ты плохой""". Публикация обоснований дает возможность заниматься физикой, а не базаром. Если не все участники дискурсии согласны с обоснованием -- это можно обсуждать. Но если обоснования нет -- обсуждать нечего. Можно, конечно, обсуждать эмпирическую формулу, метод ее получения и следствия из нее. Но отдавая себе отчет, что она эмпирическая, а не формально обоснованная.

Ну чего Вы так разнервничались? Дело в том, что когда я говорил о своем решении я не имел в виду нахождение положений зарядов при конечных $N$. Мне было достаточно тех утверждений о распределении зарядов, которые я доказал. А стоить численную схему мне было просто лень. Что касается, Ваших предположений о пределе $N \to \infty$, то может это конечно и имеет смысл с точки зрения численных методов, но с точки зрения физики Ваши размышления не имеют никакого отношения к реальности. Если оставлять заряд частиц $q$ фиксированным, то при $N\to \infty$ полный заряд стремится к бесконечности! Поэтому меня интересовал исключительно предельный переход при котором полный заряд остается конечным. В связи с этим можно сформулировать утверждение:
У т в е р ж д е н и е:
В пределе $N \to \infty$, при котором полный заряд остается конечным наступает коллапс частиц на стенки .
Решение:
Из утверждения 3 следует, что при увеличении $N$ расстояние между зарядами стремится к нулю, а значит можно ввести функцию плотности распределения зарядов формулой
$
\int_{-a}^{x_i}\rho_n(x)dx=(i-1)q
$
или $\int_{x_i}^{x_{i+1}}\rho_n(x)=q$. Записываем потенциальную энергию конечного числа частиц
$$
V=\sum_{ik} \frac{q^2}{|x_i-x_k|}
$$
условие равновесия дает
$\sum_{k=1}^{i-1} \frac{q^2}{(x_i-x_k)^2}=\sum_{k=i+1}^n \frac{q^2}{(x_k-x_i)^2}$
Наивный переход к пределу $N \to \infty$ дает расходимость в выражении
$$
\int_{-a}^x \frac{\rho(x)\rho(t)dt}{(x-t)^2}=\int_{x}^a \frac{\rho(x)\rho(t)dt}{(x-t)^2}
$$
Чтобы регуляризовать интеграл нужно исключить самодействие зарядов. В приближении непрерывной плотности, точечный заряд расположенный в окрестности точки $x$ оказывается распределен, по отрезку $(x-q/(2\rho(x)),x+q/(2\rho(x)))$. Поэтому интеграл без самодействия имеет вид
$$
\int_{-a}^{x-q/(2\rho(x))}\frac{\rho(t)dt}{(t-x)^2}=\int_{x+q/(2\rho(x))}^a\frac{\rho(t)dt}{(t-x)^2}
$$
Интегрируем по частям и получаем
$$
\frac{\rho(a)}{a-x}-\frac{\rho(-a)}{a+x}+\int_{-a}^{x-q/(2\rho(x))}\frac{\rho'(t)dt}{t-x}=
2\rho'(x)+\int_{x+q/(2\rho(x))}^a\frac{\rho'(t)dt}{t-x}
$$
Интегрируя еще раз по частям получаем
$$
-\rho'(x) -\frac{\rho(-a)}{a+x} + \rho'(x)\ln\left|t-x\right|^{x-q/(2\rho(x))}_{-a}-\int_{-a}^{x-q/(2\rho(x))}\rho''(t)\ln|t-x| dt = 
$$
$$
\rho'(x)- \frac{\rho(a)}{a-x} +\rho'(x)\ln\left|t-x\right|_{x+q/(2\rho(x))}^a-\int_{x+q/(2\rho(x))}^a \rho''(t)\ln|t-x| dt
$$
оставшиеся интегралы регулярны в пределе $q/(2\rho(x)) \to 0$ при всех $x\in (-a,a)$. Видим, что присутствует нескомпенсировангная сила, которя расталкивает заряды к стенке. В итоге имеем
$$
\rho'(x)\ln(q/(2\rho(x)))= -\rho'(x)\ln(q/(2\rho(x)))
$$
В пределе баланс сил может быть достигнут только если $\rho'(x)=0$. С другой стороны, если $\rho(x \in (-a,a))=\hbox{const} \neq 0$, то это распределение не может являться равновесным, следовательно $\rho(x)=0$. А это значит, что заряды сколлапсировали на стенки.

 Профиль  
                  
 
 
Сообщение03.06.2006, 13:33 
Экс-модератор
Аватара пользователя


30/11/05
1275
Незванный гость писал(а):
Просто свист -- это кинуть формулу без обоснования (и даже без намека на то, из чего она получена). Если Вы будете настаивать, я готов привести еще по крайней мере один пример. Вам это надо?

Ну раз уж начали вспоминать, то говорите. Лучше сразу прямо в лицо.

 Профиль  
                  
 
 
Сообщение03.06.2006, 15:27 
Заблокирован
Аватара пользователя


21/04/06

4930
Зиновий писал(а):
Шимпанзе
1. Ну и чему, по Вашему, равна электрическая емкость бесконечно тонкого диэлектрического стержня конечной длины?
2. И чему равна величина энергии запасенной в такой емкости?
P.s.
Формулы штука удобная, только, что бы правильно ими пользоваться, надо знать их "происхождение", т.е. физику, т.к. у всякой формулы есть своя "область применения", определяемая все той же физикой.


Скажите, Вы кроме как гавкать за все время пребывания на форуме ничему не научились? Молодежь тут делом занята, хотя и чистой математикой, но искренне.
Продолжайте морочить голову «голым» математикам и «круглым» химикам. Среди физиков Вы и минуты не продержитесь. Стыдно должно быть, голова ведь седая уже.
А потому «игнор»!

Шимпанзе

Шимпанзе, Вам предупреждение за переход на личности и флейм. //photon

 Профиль  
                  
 
 
Сообщение03.06.2006, 15:30 
Заслуженный участник
Аватара пользователя


18/05/06
13437
с Территории
Аурелиано Буэндиа писал(а):
Объясните, пожалуйста, почему последний (нечетный) заряд-точка должен лечь случайно?

А как же? Вот мы минимизируем целевую функцию \sum\limits_{i=0}^n \sum\limits_{j<i}f(|x_i-x_j|), где функцию f я называю потенциалом (по привычке, а может, просто слово это нравится). Если потенциал f(r)=-r, то чётное число точек ложится равными кучами по концам отрезка, а последней (нечётной) что делать? Для неё же все положения дают одно и то же значение целевой функции, нет? Вот она и ложится куда попало.
Это всё довольно скучно, а интересно рассмотреть потенциал f(r)=1/r, что мы здесь, собственно, и делаем.
Да, к слову: единственность минимума в этом случае (1/r) следует из положительной определённости второго дифференциала, а та, в свою очередь, наверное (в деталях я этого пока не вижу), следует из того, что наша потенциальная функция выпукла вниз.

 Профиль  
                  
 
 
Сообщение03.06.2006, 15:57 
Экс-модератор
Аватара пользователя


30/11/05
1275
ИСН писал(а):
А как же? Вот мы минимизируем функцию \sum\limits_{i=0}^n \sum\limits_{j<i}f(|x_i-x_j|), где функцию f я называю потенциалом (по привычке, а может, просто слово это нравится). Если функция f(r)=-r, то чётное число зарядов ложится равными кучами по концам отрезка, а последнему (нечётному) что делать? Для него же все положения дают одно и то же значение целевой функции, нет? Вот он и ложится куда попало.

В случае $f(x)=-|x_i-x_k|$ вы правы. Значит я Вас неправильно понял. Я думал Вы говорите о $1/|x_i-x_k|$. В этом случае в равновесии последний нечетный ляжет в центр.
Цитата:
Это всё довольно скучно, а интересно рассмотреть потенциал f(r)=1/r, что мы здесь, собственно, и делаем.

Общая картина для $1/|x_i-x_k|$ тоже ясна. Тут можно только детали уточнять. Намного сложнее и интереснее рассматривать непрерывную модель.

 Профиль  
                  
 
 
Сообщение03.06.2006, 17:46 
Заслуженный участник
Аватара пользователя


17/10/05
3709
:evil:
Аурелиано Буэндиа писал(а):
Ну раз уж начали вспоминать, то говорите. Лучше сразу прямо в лицо.

Извольте.

 Профиль  
                  
 
 
Сообщение03.06.2006, 18:23 
Заслуженный участник
Аватара пользователя


23/07/05
17973
Москва
Аурелиано Буэндиа писал(а):
куча народа в этой теме (Зиновий и др.) считают, что заряды сгрупируются по стенкам...


Извините, но Вы не обратили внимание, что "куча народа", включая Зиновия, решали другую задачу, в которой взаимодействие двух зарядов не зависит от расстояния между ними. Конечно, решение в этом случае очевидно, но также очевидно, что оно отличается от решения той задачи, которую рассматриваете Вы.

 Профиль  
                  
 
 
Сообщение03.06.2006, 18:51 
Экс-модератор
Аватара пользователя


30/11/05
1275
незванный гость писал(а):
Извольте.

Ну и что тут такого? Между прочим, мое первое решение в той теме было аналогично тому что Вы потом привели. Но я при решении допустил досадную ошибку в вычислениях. Поэтому не было смысла его озвучивать и я привел другое решение. Но к чему вообще этот разговор, Вы что не допускали ошибок? Могу дать ссылку, если хотите.

 Профиль  
                  
 
 
Сообщение03.06.2006, 18:52 
Экс-модератор
Аватара пользователя


30/11/05
1275
Someone писал(а):
Извините, но Вы не обратили внимание, что "куча народа", включая Зиновия, решали другую задачу, в которой взаимодействие двух зарядов не зависит от расстояния между ними. Конечно, решение в этом случае очевидно, но также очевидно, что оно отличается от решения той задачи, которую рассматриваете Вы.

Спасибо, что разъяснили. Тогда это было недоразумение.

 Профиль  
                  
 
 
Сообщение03.06.2006, 19:13 
Заслуженный участник
Аватара пользователя


17/10/05
3709
:evil:
Аурелиано Буэндиа писал(а):
Ну чего Вы так разнервничались? Дело в том, что когда я говорил о своем решении я не имел в виду нахождение положений зарядов при конечных $N$. Мне было достаточно тех утверждений о распределении зарядов, которые я доказал. А стоить численную схему мне было просто лень. Что касается, Ваших предположений о пределе $N \to \infty$, то может это конечно и имеет смысл с точки зрения численных методов, но с точки зрения физики Ваши размышления не имеют никакого отношения к реальности. Если оставлять заряд частиц $q$ фиксированным, то при $N\to \infty$ полный заряд стремится к бесконечности! Поэтому меня интересовал исключительно предельный переход при котором полный заряд остается конечным.

Оставьте, пожалуйста, заботу о моем эмоциональном состоянии. Как-то недостойно модератора. Давайте лучше о физике...

1) Где бы это я рассматривал систему при неизменном заряде частиц? Всякое мое рассуждение было либо для непрерывного разряда, либо для фиксированного и неизменявшегося количества частиц. Был, правда, вопросец о стабильности оного распределения, но он вроде никак не связан с численными экспериментами на следующий день.

2) То, что Вам достаточно доказанных Вами утверждений, просто замечательно. Хотелось бы только, чтобы Вы чаще делились своими доказательствами, а не утверждениями (оные, в отсутствии доказательств, становяться бездоказательными утверждениями). Например, Вы по прежнему не обосновали Вашу формулу для минимального расстояния. Это уже т_р_е_т_и_й запрос.

3) Высказывать утверждения о свойствах глобального минимума конечно можно. Особенно не уточняя, что он глобальный :wink:. Вопрос в их ценности. Глобальная точка равновесия одна, и потому потенциал в ней является единственным числом -- естественно, ограниченным сверху (при наличии примера конечных состояний). Но слова глобальный в своем исходном сообщении Вы нигде не произнесли... Единственность же равновесного состояния -- не обязательно глобального минимума -- по прежнему открытый вопрос.

4) Я склонен считать (на основании численных экспериментов; меня в данном случае интересует очень сильная оценка, а не просто стремление к нулю), что расстояние между заряженными частицами остается ограниченным сверху: $r_{max} < k \frac{2a}{n-1}$, $k$ не зависит от $n$. Из чего следует несколько выводов -- а) энергия системы растет как логарифм количества частиц при сохранении общего заряда системы; б) при переходе плотности плотность остается положительной функцией $\rho(x) > 1/ k$. Физически это вполне осмысленно, поскольку выражение для потенциала потеряло смысл...

5) Ваш метод исключения сингулярности меня смущает. Поскольку интеграл по выброшенной Вами окрестности остается бесконечным (со знаком). Это значит, что на заряд действует бесконечная сила, возникающая локально. В этом случае всем остальным можно просто пренебречь за конечностью. С моей точки зрения, это закрывает физически вопрос о написанном Вами потенциале системы. Но не вопрос о поведении системы -- его, вероятно, можно получить, рассматривая тонкий цилиндр.

 Профиль  
                  
 
 
Сообщение03.06.2006, 19:38 
Экс-модератор
Аватара пользователя


30/11/05
1275
незванный гость писал(а):
1) Где бы это я рассматривал систему при неизменном заряде частиц? Всякое мое рассуждение было либо для непрерывного разряда, либо для фиксированного и неизменявшегося количества частиц. Был, правда, вопросец о стабильности оного распределения, но он вроде никак не связан с численными экспериментами на следующий день.


незванный гость писал(а):
2) То, что Вам достаточно доказанных Вами утверждений, просто замечательно. Хотелось бы только, чтобы Вы чаще делились своими доказательствами, а не утверждениями (оные, в отсутствии доказательств, становяться бездоказательными утверждениями). Например, Вы по прежнему не обосновали Вашу формулу для минимального расстояния. Это уже т_р_е_т_и_й запрос.

А Вы внимательно прочитали начало сообщения. Там есть ясный ответ на Ваш вопрос. Третья степень получается при упрощении. Насчет приоритета не спорю, только из нас двоих кажется соревнуетесь только Вы. Я думал мы просто обсуждаем. Извините.
незванный гость писал(а):
3) Высказывать утверждения о свойствах глобального минимума конечно можно. Особенно не уточняя, что он глобальный :wink:. Вопрос в их ценности. Глобальная точка равновесия одна, и потому потенциал в ней является единственным числом -- естественно, ограниченным сверху (при наличии примера конечных состояний). Но слова глобальный в своем исходном сообщении Вы нигде не произнесли... Единственность же равновесного состояния -- не обязательно глобального минимума -- по прежнему открытый вопрос.

Ну не произнес. Но подразумевал. А что Вы будете это оспаривать?
незванный гость писал(а):
4) Я склонен считать (на основании численных экспериментов; меня в данном случае интересует очень сильная оценка, а не просто стремление к нулю), что расстояние между заряженными частицами остается ограниченным сверху: $r_{max} <k> 1/ k$. Физически это вполне осмысленно, поскольку выражение для потенциала потеряло смысл...

5) Ваш метод исключения сингулярности меня смущает. Поскольку интеграл по выброшенной Вами окрестности остается бесконечным (со знаком). Это значит, что на заряд действует бесконечная сила, возникающая локально. В этом случае всем остальным можно просто пренебречь за конечностью. С моей точки зрения, это закрывает физически вопрос о написанном Вами потенциале системы. Но не вопрос о поведении системы -- его, вероятно, можно получить, рассматривая тонкий цилиндр.

Ну и что тут Вас смущает? Судя по всему Вы не знакомы с перенормировками, которые широко используются в КТП. Поэтому и говорите, что это закрывает физически вопрос. Ну конечно, если рассматривать тонкий цилиндр все будет гладко...

 Профиль  
                  
 
 
Сообщение03.06.2006, 20:25 
Заслуженный участник
Аватара пользователя


17/10/05
3709
:evil:
Суммируя Ваши выступления, процитирую Буонапарте: "пишите коротко и неясно". Вы возразили, что у Вас получилось $\sim \frac{1}{n^2}$. А теперь говорите, что я спорю о приоритете. (Ваше замечание о третьей степени в начале -- вообще замечательно. Третьей степени чего и в какой формуле. Если бы Вы потрудились добавить "должно быть $\frac{a}{n^3}$" -- никаких несностей бы возникнуть не могло.) Ваши подразумевания -- изящнейший способ делать декларации. Понимай как знаешь! а если собеседник не прочитал Ваши мысли на расстоянии -- то он глубоко неправ.

Кстати, о приоритете -- заметили ли Вы, что мое сообщение суммировало понятную мне часть дискурсии на тот момент? Я не претендовал на новизну тамошних утверждений...

А с перенормировками я незнаком. Как и с КТП. И не скрываю этого (и не строю новых теорий поля). Поэтому метод меня смущает -- я не говорю, что он неверен. А вот Ваш результат смущает меня еще больше -- с перенормировками и без. Поскольку я уверен (возможно, ошибочно) в существовании гладкого распределения заряда. И мне проще отказаться от метода вычисления потенциала... Какой-то странноватенький результат. С повседневным опытом плохо согласуется. А перенормировки редко используются в классической физике, к каковой и относится эта задача. Может, поэтому и незнаком.

 Профиль  
                  
 
 
Сообщение03.06.2006, 20:50 
Экс-модератор
Аватара пользователя


30/11/05
1275
незванный гость писал(а):
Суммируя Ваши выступления, процитирую Буонапарте: "пишите коротко и неясно"... Ваши подразумевания -- изящнейший способ делать декларации. Понимай как знаешь! а если собеседник не прочитал Ваши мысли на расстоянии -- то он глубоко неправ.

На мой взгляд, у Вас странное отношение к чужим постам. При рассмотрении задач, которые появляются на форуме я очень редко цепляюсь к чужим постам, поскольку для меня более важно, то что я решаю и то, как я понимаю тот или иной вопрос. Чужие посты являются для меня самым последним ориентиром при рассмотрении проблемы. Судя, по Вашей реакции, которая длится уже второй день, у Вас другой взгляд на этот счет. Ну хорошо, учту.
незванный гость писал(а):
Кстати, о приоритете -- заметили ли Вы, что мое сообщение суммировало понятную мне часть дискурсии на тот момент? Я не претендовал на новизну тамошних утверждений...

Ну не претендовали и ладно...
незванный гость писал(а):
И мне проще отказаться от метода вычисления потенциала... Какой-то странноватенький результат. С повседневным опытом плохо согласуется.

Ну для этого я его и вывел, чтобы обсудить. Возможно я в чем то и неправ. Сам подход к решению через тонкий цилиндр в идеологическом плане для меня тривиальный и не представляет интереса. То что было интересно я написал. Кстати, я не очень понял, что вы имели хотели сказать фразой "плохо согласуется c повседненным опытом" (Наполеона цитировать не буду).

 Профиль  
                  
 
 Формула Ньютона - Лейбница
Сообщение03.06.2006, 22:32 
Заслуженный участник
Аватара пользователя


23/07/05
17973
Москва
Зиновий писал(а):
А то, опять получится как с формулой Ньютона- Лейбница...


Я уж думал, Вы про неё забыли. Придётся объяснять, хоть и надеялся, что сами разберётесь.

Рассмотрим случай, когда плотность заряда и, следовательно, создаваемое им электростатическое поле зависят только от координаты $x$, а от $y$ и $z$ не зависят. Обозначим $\rho(x)$ плотность электрического заряда, $\vec D(x)$ - вектор электрического смещения. В силу симметрии должно быть $\vec D(x)=D(x)\vec\imath$, где, как обычно, $\vec\imath$ - орт оси $Ox$. Запишем формулу Остроградского - Гаусса:
$$\iiint\limits_W\rho(x)dxdydz=\iint\limits_{\Pi}\text{пр}_{\vec n}\vec D(x)dS\text{,}\eqno(1)$$
где $\Pi$ - внешняя сторона поверхности, ограничивающей область $W$.
В качестве области $W$ мы возьмём цилиндр, образующая которого параллельна оси $Ox$, а основания лежат в плоскостях $x=a$ и $x=b$, где $a<b$. Обозначим эти основания, соответственно, $D_a$ и $D_b$; кроме того, обозначим $D$ проекцию $W$ на плоскость $Oyz$. Преобразуем интеграл в левой части:
$$\iiint\limits_W\rho(x)dxdydz=\int\limits_a^b\rho(x)dx\iint\limits_Ddydz=S\int\limits_a^b\rho(x)dx$$,
где $S$ - площадь области $D$.
Для вычисления интеграла в правой части заметим прежде всего, что на боковой поверхности цилиндра $\text{пр}_{\vec n}\vec D(x)=\text{пр}_{\vec n}D(x)\vec\imath=0$, так как $\vec n\perp\vec\imath$, и, следовательно, интеграл по боковой поверхности цилиндра равен $0$. Кроме того, на основании $D_b$ имеем $\text{пр}_{\vec n}\vec D(x)=D(b)$, а на основании $D_a$ - $\text{пр}_{\vec n}\vec D(x)=-D(a)$. Поэтому
$$\iint\limits_{\Pi}\text{пр}_{\vec n}\vec D(x)dS=\iint\limits_{D_b}\text{пр}_{\vec n}\vec D(x)dS+\iint\limits_{D_a}\text{пр}_{\vec n}\vec D(x)dS=\iint\limits_DD(b)dydz-\iint\limits_DD(a)dydz=SD(b)-SD(a)$$.
Таким образом, формула (1) принимает вид
$$S\int\limits_a^b\rho(x)dx=SD(b)-SD(a)\text{,}\eqno(2)$$
или, после сокращения на $S$,
$$\int\limits_a^b\rho(x)dx=D(b)-D(a)\text{.}\eqno(3)$$
Ну, вот Вам и формула Ньютона - Лейбница.

Формула (3), собственно говоря, не годится для электростатики одномерного пространства, потому что плотность заряда $\rho(x)$ имеет размерность $\frac{\text{Кл}}{\text{м}^3}$, в то время как в одномерном пространстве эта размерность должна быть $\frac{\text{Кл}}{\text{м}}$. Если мы не будем сокращать произвольный множитель $S$ в формуле (2), а вместо этого фиксируем раз и навсегда величину $S=1\text{м}^2$ (это соответствует выбору единицы измерения заряда в одномерном пространстве такой же, какой она у нас была в трёхмерном), то $\gamma(x)=S\rho(x)$ будет линейной плотностью заряда; произведение $SD(x)$ я опять обозначу $D(x)$ (это будет электрическое смещение в одномерном пространстве; как видите, размерность смещения в одномерном и в трёхмерном пространстве разная). Тогда формула (2) превратится в
$$\int\limits_a^b\gamma(x)dx=D(b)-D(a)\text{,}\eqno(4)$$
но, в отличие от формулы (3), размерности всех величин здесь соответствуют одномерному пространству, а не трёхмерному.

Не нужно сильно удивляться тому, что размерности некоторых величин в пространствах разной размерности могут не совпадать. Сравним, например, закон Кулона с соответствующим законом для одномерного пространства (в скалярной форме): $F=\frac{q_1q_2}{4\pi\varepsilon\varepsilon_0r^2}$ и $F=\frac{q_1q_2}{2\varepsilon\varepsilon_0}$. Поскольку относительная диэлектрическая проницаемость $\varepsilon$ безразмерна, то, если мы хотим сохранить неизменными размерности силы и электрического заряда, придётся согласиться с тем, что размерность диэлектрической проницаемости вакуума $\varepsilon_0$ в этих формулах различна. Так как $F=qE$, то размерность напряжённости электрического поля в обоих случаях одна и та же, а вот размерность электрического смещения $D=\varepsilon\varepsilon_0E$ будет отличаться, причем, именно так, как она отличается в формулах (3) и (4).

Зиновий писал(а):
Вектор D равен потоку вектора D деленному на площадь.
Площадь, в одномерном случае равна нулю.
Вектор D стремится к бесконечности.


Это всё хорошо, но только непонятно, на каких основаниях Вы считаете нульмерную "площадь" (математики предпочитают здесь слово "объём") нульмерной сферы равной $0$. Нульмерная "площадь" есть безразмерная величина. Естественно считать, что нульмерная площадь множества равна числу точек в этом множестве. Нульмерная сфера - это граница одномерного шара, то есть, отрезка. Она состоит из двух точек, поэтому её нульмерная "площадь" равна $2$. Поэтому делить нужно на $2$, а не на $0$. И это прекрасно согласуется как с формулой (4), так и с формулой напряжённости электростатического поля заряженной плоскости: $E=\frac{\sigma}{2\varepsilon\varepsilon_0}$.

P.S. Очень уж у меня буква $D$ оказалась перегруженной...

 Профиль  
                  
Показать сообщения за:  Поле сортировки  
Начать новую тему Ответить на тему  [ Сообщений: 250 ]  На страницу Пред.  1 ... 4, 5, 6, 7, 8, 9, 10 ... 17  След.

Модераторы: photon, whiterussian, profrotter, Jnrty, Aer, Парджеттер, Eule_A, Супермодераторы



Кто сейчас на конференции

Сейчас этот форум просматривают: нет зарегистрированных пользователей


Вы не можете начинать темы
Вы не можете отвечать на сообщения
Вы не можете редактировать свои сообщения
Вы не можете удалять свои сообщения
Вы не можете добавлять вложения

Найти:
Powered by phpBB © 2000, 2002, 2005, 2007 phpBB Group